Need help understanding something in my proof

  • Thread starter Samuelb88
  • Start date
  • Tags
    Proof
In summary: S_1 from? It's not part of the original problem. Here is the proof that works:Given E>0 we want to show that there is an N such that for all n > N, |S_n| < E.From the problem statement, we know that 0 < S_n < r S_{n-1} for all n. This means that 0 < S_n < r S_{n-1} < r^{2}S_{n-2} < \cdots < r^{n-1} S_{1} for all n.Now, for all n > m , 0 < S_n
  • #1
Samuelb88
162
0

Homework Statement


Suppose [tex]S_n > 0[/tex] and [tex]r S_n > S_{n+1}[/tex], where r is a constant such that 0<r<1. Show that [tex] \lim_{n\rightarrow +\infty} {S_n} = 0[/tex].

The Attempt at a Solution



First, I'd like to know if my proof is correct.

Proof: Suppose [tex]S_n > 0[/tex] and [tex]r S_n > S_{n+1}[/tex], where r is a constant such that 0<r<1. First note [tex]S_n[/tex] is decreasing since [tex]S_n > S_{n+1}[/tex] if [tex]r S_n > S_{n+1}[/tex]. Moreover, for every n [tex]S_n > 0[/tex], so [tex]r S_n > S_{n+1} > 0[/tex]. Therefore the [tex] \lim_{n\rightarrow +\infty} {S_n} = 0[/tex].

Suppose my proof is correct. Then my question is what role does r play in this problem? It seems to me, so long as [tex]S_n[/tex] is decreasing, and [tex]S_n > S_{n+1}[/tex], and for every n [tex]S_n > 0[/tex], then the [tex] \lim_{n\rightarrow +\infty} {S_n} = 0[/tex]. Right?
 
Physics news on Phys.org
  • #2
Your proof is not correct, S_n could be decreasing but the limit could non-zero. The fact that it is bounded below by zero doesn't imply it approaches zero. Example: consider the sequence 1/2 + 1/n. Cleary, it is decreasing and bounded below by zero but the limit is 1/2.

I think you have to show that r^n goes to zero and since S_n is bounded then the sequence S_n goes to zero since we can make S_n as small as we like.
 
  • #3
The problem says for all n rSn>Sn+1. This isn't a proof but it leads to it. Say r is .5 for example. Then .5 > Sn+1/Sn. It seems easy to derive a contradiction when lim of Sn is non zero. If you keep taking halves of Sn, you will go lower than your non zero limit. But if Sn is approaching zero, then you can easily keep taking halves and never even touch zero. Now just generalize.
 
Last edited:
  • #4
I am still a little confused about how to conclude that the [tex] \lim_{n\rightarrow +\infty} {S_n} = 0[/tex]. Here is what I've come up with:

Suppose for every n, [tex]S_n > 0 [/tex] and for some fixed r, 0<r<1, [tex]r S_n < S_{n+1}[/tex]. Let r=1/(1+a), a > 0. Then [tex]S_n[/tex] is a decreasing sequence, and is bounded below. Then for every n

[tex] 0 < \frac{S_{n+1}}{S_n} < \frac{1}{1+a}[/tex].

We can arbitrarily make 1/(1+a) as close to zero as we want by choosing an extremely large a, and the inequality will still hold. In other words, I can squeeze [tex]S_{n+1} / S_n[/tex] between 0 and 1/(1+a) << 1.

At this point, may I conclude [tex] \lim_{n\rightarrow +\infty} {S_n} = 0 [/tex] since the ratio of [tex]S_{n+1}[/tex] and [tex]S_n[/tex] can be squeezed between 0 and a very small number, the sequence [tex]S_n[/tex] is decreasing and is bounded below by zero?
 
  • #5
The point is, Sn will go to zero if r is .99999999 or .00000001 and you cannot choose r, it is some given value between 0 and 1. Think about what (r=.5) .5>Sn+1/Sn means. It means at the least, Sn+1 is 2 times smaller than Sn+1 FOR ALL n.
 
  • #6
Try finding a bound for S_n in terms of n .(Hint: Try induction.)
 
  • #7
Samuelb88 said:
I am still a little confused about how to conclude that the [tex] \lim_{n\rightarrow +\infty} {S_n} = 0[/tex]. Here is what I've come up with:

Suppose for every n, [tex]S_n > 0 [/tex] and for some fixed r, 0<r<1, [tex]r S_n < S_{n+1}[/tex]. Let r=1/(1+a), a > 0. Then [tex]S_n[/tex] is a decreasing sequence, and is bounded below. Then for every n

[tex] 0 < \frac{S_{n+1}}{S_n} < \frac{1}{1+a}[/tex].

We can arbitrarily make 1/(1+a) as close to zero as we want by choosing an extremely large a, and the inequality will still hold. In other words, I can squeeze [tex]S_{n+1} / S_n[/tex] between 0 and 1/(1+a) << 1.

At this point, may I conclude [tex] \lim_{n\rightarrow +\infty} {S_n} = 0 [/tex] since the ratio of [tex]S_{n+1}[/tex] and [tex]S_n[/tex] can be squeezed between 0 and a very small number, the sequence [tex]S_n[/tex] is decreasing and is bounded below by zero?

The way I envisioned the proof was like this

[tex] rS_0 > S_1[/tex]
[tex] r^{2}S_0 > S_2[/tex]
.
.
.
[tex] r^{n}S_{0} > S_{n}[/tex]

Using the definition of a limit we want to show that [itex] S_n \rightarrow 0[/itex]

[tex] 0< | S_{n} -0 | < |S_{0}|r^{n} [/tex] . We know [itex] S_0[/itex] is bounded since the original sequence is convergent.

If you have already proving that [itex] \lim_ {n \to \infty } r^{n} = 0 \quad 0<r<1[/itex] then the proof is complete.
 
  • #8
╔(σ_σ)╝ said:
The way I envisioned the proof was like this

[tex] rS_0 > S_1[/tex]
[tex] r^{2}S_0 > S_2[/tex]
.
.
.
[tex] r^{n}S_{0} > S_{n}[/tex]

Using the definition of a limit we want to show that [itex] S_n \rightarrow 0[/itex]

[tex] 0< | S_{n} -0 | < |S_{0}|r^{n} [/tex] . We know [itex] S_0[/itex] is bounded since the original sequence is convergent.

If you have already proving that [itex] \lim_ {n \to \infty } r^{n} = 0 \quad 0<r<1[/itex] then the proof is complete.

Okay, for the most part, I understand what you are saying here. (Didn't understand how you derived the relation between [tex]r^n S_1[/tex] and [tex] S_n[/tex] originally... :)

How about this:

We want to show using the definition of a limit that [tex] S_n \rightarrow 0 [/tex] if for ever E >0, there is an N s.t. whenever n>N, [tex] |S_n| < E[/tex]. Let E >0. Suppose [tex]S_n > 0 [/tex] (bounded) and [tex] r S_n > S_{n+1} [/tex] (decreasing). So

[tex] rS_1 > S_2[/tex]
[tex] r^{2}S_1 > S_3[/tex]
.
.
.
[tex] r^{n}S_{1} > r^{n-1} S_{1} > S_{n}[/tex] which is true for all n.

Let [tex] E= r^{n} S_1[/tex]. As required. Since [tex]S_n < r^n S_1[/tex] for all n, and [tex]r^n \rightarrow 0[/tex] as [tex] n \rightarrow +\infty[/tex], then the [tex] \lim_{n\rightarrow _\infty} {S_n} = 0[/tex]. ?
 
  • #9
Samuelb88 said:
Let [tex] E= r^{n} S_1[/tex]. As required. Since [tex]S_n < r^n S_1[/tex] for all n, and [tex]r^n \rightarrow 0[/tex] as [tex] n \rightarrow +\infty[/tex], then the [tex] \lim_{n\rightarrow _\infty} {S_n} = 0[/tex]. ?
No, you can't just set [tex] \epsilon = r^{n} S_0 [/tex] [URL]http://i236.photobucket.com/albums/ff286/nfforums/NF%20smilies/x31pt9.gif[/URL]

The definition of a limit requires that for [tex] n > n_{0} [/tex] the limit holds and [tex] n_0[/tex] should depend of epsilon.

Also [tex] 0 < S_n < r^n S_1[/tex] the 0 < part is important since you want [tex] S_n[/tex] to be sandwiched by zero from the left and right.
You want to pick an [tex] n_0[/tex] such that for [tex] n > n_{0} [/tex] you can make tex] r^n S_1[/tex] as small as possible.

[tex] \lim_{n\rightarrow _\infty} {S_n} = 0[/tex].

I am not sure if you have to prove this fact or you can just use it .

If you are simply allowed to use it then you can just state that since r^n goes to zero then epsilon and n_0 exist. Or you could jsimply use the squeeze theorem/ sandwich lemma.
 
Last edited by a moderator:
  • #10
╔(σ_σ)╝ said:
Also [tex] 0 < S_n < r^n S_1[/tex] the 0 < part is important since you want [tex] S_n[/tex] to be sandwiched by zero from the left and right.
You want to pick an [tex] n_0[/tex] such that for [tex] n > n_{0} [/tex] you can make tex] r^n S_1[/tex] as small as possible.

Okay, so we agree that for every n, [tex] 0 < S_n < r^{n} S_{1} [/tex]. And from my book, there is an example which proves that for every a such that 0<a<1, the [tex] \lim_{n\rightarrow +\infty} {a^n} = 0[/tex]. So I may use that fact in my proof. I was thinking that since we know when 0<r<1 that [tex]r^n \rightarrow 0[/tex] as [tex] n \rightarrow +\infty[/tex], then we know for every ε > 0, there must exist an N such that whenever n > N, [tex] | r^n | < \epsilon[/tex]. So couldn't we just arbitrarily choose an n > N (since we know such N exists) such that [tex] S_1 |r^n| < \epsilon[/tex]. Since 0<r<1, and for every n, [tex] S_1 > 0[/tex], then [tex] S_1 |r^n| = S_1 r^n[/tex], and therefore we have found such n such that [tex] 0 < S_n < r^n S_1 < \epsilon [/tex].
I am not sure if you have to prove this fact or you can just use it .

If you are simply allowed to use it then you can just state that since r^n goes to zero then epsilon and n_0 exist. Or you could jsimply use the squeeze theorem/ sandwich lemma.

I am pretty sure I can not use the fact that [tex] \lim_{n\rightarrow +\infty} {S_n} = 0[/tex], that is, that's what we're trying to prove. At any rate, I was thinking about using the squeeze theorem, but I thought the squeeze theorem required less than or equal to signs, and we only have less than signs in our inequality. Other wise I would be squeezing the limit of [tex] S_n [/tex] between some greater than zero, and less than zero, which to me seems invalid. Can you clarify this for me, please? :confused:
 
  • #11
You can use the fact that [itex]0< S_n< r^nS_1[/itex], to argue that [itex]|S_n- 0|= |S_n|< |S_1|r^n< \epsilon[/itex] implies that [itex]r^n< \epsilon/|S_1|[/itex]. How large does n have to be in order that that be true?
 
  • #12
HallsofIvy said:
You can use the fact that [itex]0< S_n< r^nS_1[/itex], to argue that [itex]|S_n- 0|= |S_n|< |S_1|r^n< \epsilon[/itex] implies that [itex]r^n< \epsilon/|S_1|[/itex]. How large does n have to be in order that that be true?

Great. I would have to choose n to be an integer greater than [tex] ln ( \frac{( \epsilon }{|S_1| }) / ln(r)[/tex]. This choice for n makes sense because 0<r<1, ε > 0, and [tex] |S_1| > 0 [/tex].

Edit: Oops, forgot to take ln on other side. :D
 
Last edited:
  • #13
Samuelb88 said:
Great. I would have to choose n to be an integer greater than [tex] ln ( \frac{( \epsilon }{|S_1| }) / ln(r)[/tex]. This choice for n makes sense because 0<r<1, ε > 0, and [tex] |S_1| > 0 [/tex].

Edit: Oops, forgot to take ln on other side. :D
Good. You are basically done. Just everything together.
 
  • #14
Btw your inequality is wrong.
[tex] S_n < r^{n}S_0[/tex] is correct if you start from n=0.

But since you are starting for n=1 your inequality should be...
[tex] S_n < r^{n-1}S_1 [/tex].

You made a mistake when you said... [tex] r^{n-1}S_1 < r^{n}S_1[/tex]. It is not true since 0< r<1.

This is just a minor correction since [tex] r^{n-1} \rightarrow 0[/tex].
 
  • #15
Suppose [tex] 0 < S_n[/tex] and [tex] S_{n+1} < r S_n [/tex] where r is a constant such taht 0<r<1. Then [tex] S_n < r^{n} S_0[/tex].

Lemma: From an example in my book, we know when 0<a<1 that the [tex]lim_{n\rightarrow +\infty} {a^n} = 0[/tex]. That is, for every ε > 0, there is an N such that whenever n>N, [tex] |a^n| < \epsilon[/tex].

Note that the sequence is bounded below, since for every n, [tex] 0 < S_n[/tex]. Then for every n, [tex] 0 < S_n < S_0 r^n [/tex] which implies for some n>N, [tex] 0 < |S_n| < |S_0| r^n < \epsilon [/tex], which implies [tex] r^n < \epsilon/|S_0|[/tex]. Observe that [tex]0 < r, \epsilon, |S_0|[/tex], so take [tex] N = ln( \epsilon/|S_0|) / lnr [/tex], and choose n>N. Then [tex]|S_n - 0| < \epsilon[/tex] and therefore [tex] lim_{n\rightarrow +\infty} {S_n} = 0[/tex]. Done!

How does this look?
 
  • #16
Samuelb88 said:
Then for every n, [tex] 0 < S_n < S_0 r^n [/tex] which implies for some n>N, [tex] 0 < |S_n| < |S_0| r^n < \epsilon [/tex], which implies [tex] r^n < \epsilon/|S_0|[/tex]. Observe that [tex]0 < r, \epsilon, |S_0|[/tex], so take [tex] N = ln( \epsilon/|S_0|) / lnr [/tex], and choose n>N. Then [tex]|S_n - 0| < \epsilon[/tex] and therefore [tex] lim_{n\rightarrow +\infty} {S_n} = 0[/tex]. Done!

How does this look?

This part is not too good.

[tex] 0 < |S_n| < |S_0| r^n < \epsilon [/tex], which implies [tex] r^n < \epsilon/|S_0|[/tex].

This part is not true.

What is true is that ...
[tex] \lim_{n \to \infty} r^{n} = 0[/tex] implies that there exist [tex]N_0[/tex] such that for [tex] n > N_{0}[/tex],[itex] r^{n} < \frac{\epsilon}{|S_0|}[/itex].The rest of the proof is okay.:wink:
 
  • #17
Okay, now I see how I am suppose to prove the limit is zero. Thanks much, all of you! :)
 

1. What should I do if I don't understand a concept in my proof?

If you don't understand a concept in your proof, the first step is to try approaching the problem from a different angle. Look for alternative explanations or visual aids that can help you understand the concept better. If you still have trouble, don't be afraid to reach out for help from a teacher, colleague, or online community.

2. How can I identify where I am going wrong in my proof?

To identify where you may have made a mistake in your proof, it can be helpful to break the proof down into smaller steps and check each step for errors. You can also try working backwards from the conclusion to see if you can arrive at the starting point. Another useful strategy is to have someone else review your proof and provide feedback.

3. How can I improve my understanding of proofs?

One way to improve your understanding of proofs is to practice regularly. Start with simpler proofs and gradually work your way up to more complex ones. You can also read and study proofs from textbooks or online resources to gain a better understanding of different proof techniques. Additionally, seeking guidance from a mentor or tutor can also be beneficial.

4. How do I know if my proof is correct?

To determine if your proof is correct, you can check if it follows the logical flow and if every step is supported by evidence. It's also important to double check your calculations and make sure they are accurate. If you are unsure, you can always seek feedback from others or consult with a teacher or mentor.

5. How can I make my proofs more clear and concise?

To make your proofs more clear and concise, it's important to use precise and concise language, avoid unnecessary steps or explanations, and organize your proof in a logical and easy-to-follow manner. You can also use diagrams or visual aids to help illustrate your proof. Practice and seeking feedback from others can also help improve the clarity and conciseness of your proofs.

Similar threads

  • Calculus and Beyond Homework Help
Replies
11
Views
1K
  • Calculus and Beyond Homework Help
Replies
7
Views
1K
  • Calculus and Beyond Homework Help
Replies
7
Views
1K
  • Calculus and Beyond Homework Help
Replies
7
Views
1K
  • Set Theory, Logic, Probability, Statistics
Replies
6
Views
1K
  • Set Theory, Logic, Probability, Statistics
Replies
2
Views
1K
  • Set Theory, Logic, Probability, Statistics
Replies
8
Views
1K
  • Calculus and Beyond Homework Help
Replies
2
Views
711
  • Calculus and Beyond Homework Help
Replies
6
Views
1K
  • Calculus and Beyond Homework Help
Replies
2
Views
4K
Back
Top